Added topic tags to some S&J-4 problems + minor typos
[course.git] / latex / problems / Serway_and_Jewett_4 / problem24.22.tex
1 \begin{problem*}{24.22} % power <-> EM fields
2 An AM radio station broadcasts isotropically (equally in all
3 directions) with an average power of $4.00\U{kW}$.  A dipole recieving
4 antenna $65.0\U{cm}$ long is at a location $4.00\U{miles}$ from the
5 transmitter.  Compute the amplitude of the emf that is induced by this
6 signal between the ends of the recieving antenna.
7 \end{problem*}
8
9 \begin{solution}
10 To find the signal intensity at our antenna, we note that the power
11 broadcast from the station is spread out over a sphere of radius
12 $R=4.00\U{miles}$.  The average intensity is then
13 \begin{equation}
14   I = S_\text{avg} = \frac{P}{A} = \frac{P}{4\pi R^2}
15     = \frac{4.00\E{3}\U{W}}{4\pi(4.00\U{miles}\cdot 1.609\E{3}\U{m/mile})^2}
16     = 7.68\U{$\mu$W/m$^2$} \;.
17 \end{equation}
18 From Equation 24.27, we see
19 \begin{align}
20   S_\text{avg} &= \frac{E_\text{max}^2}{2\mu_0 c} \tag{24.27} \\
21   E_\text{max} &= \sqrt{2\mu_0 c S_\text{avg}} = 76.1\U{mV/m}
22 \end{align}
23 The total voltage difference produced across our length $L=65.0\U{cm}$
24 antenna is then
25 \begin{equation}
26   \Delta V = LE_\text{max} = \ans{49.4\U{mV}}
27 \end{equation}
28 \end{solution}